HW 1

You might also like

You are on page 1of 5

Math 6510 Homework 1

Tarun Chitra
February 3, 2011
1
Let X be a topological space and suppose that x
0
, x
1
are in the same path component. Since they are in the same
component, there exists a (continuous) path h : I X such that h(0) = x
1
, h(1) = x
0
. Let T be the set of homotopy
classes of paths from x
0
to x
1
and dene F : T
1
(X, x
0
) by F([f]) = [f h]. This is a loop based at x
0
and as such
[f h]
1
(X, x
0
). We need to show that this map is well-dened and bijective.
Well-Dened:
Suppose f g or in other words [f] = [g] T. If h
t
is the homotopy between f, g and 1
t
is the identity homotopy (i.e.
1
t
(x) = x, t) then h
t
1
t
gives a homotopy between f h and g h.
Injectivity:
Suppose that [f h] = [g h] in
1
(X, x
0
). Then [f h] [

h] = [f] and [g h] [

h] so that [f] = [g].


Surjectivity:
Let [f]
1
(X, x
0
). Note that [f

h] is a path between x
0
and x
1
so that it makes sense to consider F([f

h]). It is
clear that F([f

h]) = [f].
2
Let X be path connected and suppose that h is the path between x
0
and x
1
. Let
h
:
1
(X, x
0
)
1
(X, x
1
) be
the change of basepoint homomorphism,
h
([f]) = [h f

h]. Suppose that g h. Then we want to show that

h
([f]) =
g
([f]), f
1
(X, x
0
). Let H
t
be the homotopy between g, h. Then H
t
1
t


H
t
is a homotopy between
[h f

h] and [g f g]. Hence


h
=
g
, h g.
3
Let X be a path connected space.
() Suppose that
1
(X, x
0
) is Abelian for all x
0
X. From problem 1 we know that T(x
0
, x
1
) = [h] : h : I X, h(0) =
x
0
, h(1) = x
1
is in bijective correspondance with
1
(X, x
0
) for any x
1
X. Since we already know that the change of
basepoint homomorphism is a well-dened map, we can without the loss of generality choose a representative from each
class in
1
(X, x
0
) that corresponds to a class in T(x
0
, x
1
). Now suppose that [g], [h] T, so that [h g]
1
(X, x
0
).
To reiterate, the result of problem 1 states that every element of
1
(X, x
0
) is represented by such a loop, so this is
equivalent to picking an arbitary loop in
1
(X, x
0
). We want to show that for arbitrary
1
[f], [k] T(x
0
, x
1
) that

f
([h g]) =
k
([h g]). The proof is more easily explained upon considering the following drawing:
x
0
x
1
f g
h k
1
The result of problem two justies looking at the homotopy classes in P as opposed to themselves
1
Math 6510
Professor Hatcher
Homework 1
Net ID: tc328
Tarun Chitra
February 3, 2011
Now note the following:

f
([h g]) = [

f h g f] = [(

f h) ( g f)] = [( g f) (

f h)] = [ g h] (1)
= [ g k

k h] = [( g k) (

k h)] = [

k h g k] (2)
=
k
([h g]) (3)
The 3rd equalities of (1), (2) follow from the fact that
1
(X) is Abelian. This shows that the change of basepoint
homomorphism is independent of path chosen.
() Suppose that the basepoint-change homomorphisms
h
only depend on the endpoints of the path h. This means
that if h, g are paths between x
0
, x
1
X, then
h
([f]) =
g
([f]), f
1
(X, x
0
). Now suppose that x
1
= x
0
, for then
h, g are simply loops in
1
(X, x
0
). Now let f, g
1
(X, x
0
) and note the following:
[g] = [g g g] =
g
([g]) =
f
([g]) = [f g

f]
If we multiply on the left by [f] we have [g] [f] = [g f] = [f g

f] [f] = [f g

f f] = [f g] which shows that
1
(X, x
0
)
is Abelian. Since X is path connected, this means that
1
(X, x
0
)

=
1
(X, x
1
), x
1
X.
Alternatively, note that we can show this rather quickly by using problem 1. Since the basepoint-change homo-
morphism only depends on endpoints, note that if h is a path from x
0
to x
1
and g is a path from x
1
to x
0
, then
[h g]
1
(X, x
0
). Since the change of basepoint homorphism is endpoint indpendent,
h
=
g
. This gives an explicit
homotopy between h g and g h, so that from the bijection found in problem 1,
1
(X) is necessarily Abelian.
5
We will prove the following implications in order to arrive at (a) (b) (c). (a) = (b):
Suppose that every map S
1
X is homotopic to a constant map, with image a point. Suppose f : S
1
X and let the
homotopy be H : S
1
xI X so that H[
S
1
{0}
= f, H[
S
1
{1}
= x
0
for some x
0
X. Now parametrize D
2
by a radial
coordinate r [0, 1] and an angular coordinate [0, 2) (which will serve as a well-dened angle form on S
1
) and
dene a map F : D
2
X, F(r, ) = H(, 1 r). This map is continuous since H is continuous and dened on all of
D
2
. Finally, note that F(1, ) = f(), so that F restricts to f on D
2
.
(b) = (a):
Suppose that every map S
1
X extends to a map D
2
X. Let f : S
1
X be a continuous map and

f : D
2
X
its extension. Since D
2
is a contractible, there exists map : D
2
x
0
and : x
0
D
2
such that 1l
x0
and
1l
D
2. As such, composing

f with gives a homotopy H between

f and a constant map g : D
2
X, g(D
2
) =

f(x
0
).
For instance, if D
2
R
2
and we choose x
0
= 0, then this homotopy is simply H(x, t) =

f(tx). Since

f[
D
2 = f, this
gives a homotopy between f and a constant map. In the case of D
2
R
2
, this homotopy is explicitly given by simply
restricting H : D
2
I X to D
2
I or in other words H(x, t) =

f(tx), [x[ = 1.
(b) = (c):
Note that every map S
1
X is a loop and that if a map f : S
1
X extends to a map

f : D
2
X, then there is
a homotopy between f,

f. Using the argument of the previous implication, (b) = (a), there is a homotopy between

f and a constant map. Since the binary relation "is homotopic to" is transitive, this implies that f is homotopic to a
constant map. Since all of the above homotopies will preserve basepoint, this is a loop homotopy. Hence [f] is trivial
and
1
(X, x
0
) = 0, x
0
since the base of the loop and the choice of loop were arbitrary.
(c) = (a):
Suppose that
1
(X, x
0
) = 0, x
0
X. The denition of a loop in a space X based at x
0
is a continuous map : I X
such that (0) = (1) = x
0
. This is equivalent to a map : S
1
X since we can compose with the quotient
map p : I S
1
that maps p(0) = p(1). Hence the loop space of X, denoted [S
1
, X] is the set of homotopy classes
of all maps S
1
X. Since
1
(X, x
0
) is trivial for all x
0
X this implies that [S
1
, X] is necessarily trivial. One
could form this category theoretically by consider the category 1 of homotopic loops in X. Within this category, we
see that the above argument implies that [S
1
, X]

= [h]
1
(X, x
0
), for some x
0
X under the "loop morphism"
[f] [S
1
, X] [f, f(

)]. Note that this isomorphism, given the hypotheses of this problem is only valid because of the
triviality conditions in (a), (c). In the next problem, we will see that the same result is true, without such a triviality
condition, if X is path-connected by using the inverse of the above loop morphism, a forgetful-like functor.
6
Let (X, x
0
) be a path-connected, pointed space, dene [S
1
, X] =

[f] : f : S
1
X, f cts

and dene the map :

1
(X, x
0
) [S
1
, X] by [f, x
0
] [f] (i.e. forget the basepoint x
0
). The idea is that [S
1
, X] is the set of equivalence
classes of maps S
1
X under a homotopy of maps whereas
1
(X, x
0
) is the set of equivalence classes of maps S
1
X
under homotopies of loops (i.e. homotopic relative to a distinguished point). As the map is the forgetful functor
2
Math 6510
Professor Hatcher
Homework 1
Net ID: tc328
Tarun Chitra
February 3, 2011
(inclusion) from the category of pointed topological spaces,

Top to the category of topological spaces Top, the result of
this problem depicts that for path-connected spaces, the spaces of map homotopies and loop homotopies are in one-to-one
correspondance. Now we have to show two things:
is surjective
is injective on conjugacy classes, i.e. ([f]) = ([g]) h
1
(X, x
0
) [hf

h] = [g]
Surjectivity:
Let [f] [S
1
, X] and let be the standard angle form on S
1
and let

[0, 2) be an arbitrary angle. Since X


is path-connected, there is a path h between f(

) and the basepoint x


0
. Then the map hf

h is a loop based at x
0
that corresponds to [f] under . We still need to establish that the map is well-dened, since the class [f] [S
1
, X]
is only dened up to a homotopy of maps. Suppose that f, g [f], homotopic via a map L : S
1
I X such that
L[
S
1
{0}
= f, L[
S
1
{1}
= g. Let h,

h be paths between x
0
and f(

), g(

). Then we need to show that hf

h h

h
is a homotopy of loops. Since X is path-connected, a path h
t
between x
0
and H(

, t), t I, so dene the map


H : X I X by H[
X{t}
= h
t
. This is a continuous map (since the path h
t
is a continuous map for all t) and as
such denes a homotopy between h, h

. Then the map H


t
L
t


H
t
, where is the group operation within
1
denes a
homotopy between hf

h and h

h.
is surjective
Injectivity on Conjugacy Classes:
() Suppose that ([f]) = ([g]). This means that as maps, f g via a homotopy H. Since f, g are loops based at
x
0
, we need to ensure that H is a homotopy of loops. This means that if f(

) = g(

) = x
0
then we need to construct a
loop such that H
t
(

) = x
0
, t, where H
t
= H[
X{t}
. Since X is path-connected, let h be a path between x
0
and f(

).
Then h f

h is a loop based at x
0
that is loop homotopic to g.
() Now suppose that there exists an [h]
1
(X, x
0
) such that hf

h g. We want to show that ([f]) = ([g]) or


in other words, f g as a homotopy of maps. Now note that as a loop, h : [0, 1] X, h(0) = h(1) = x
0
is homotopic
(as a map) to the constant map at x
0
via the homotopy H : I I X, H(x, t) = h(tx). Hence hf

h is homotopic as a
map to f by using the aforementioned homotopy H and

H.
7
A picture of the map f : S
1
I S
1
I, f(, s) = ( + 2s, s) for a xed is the following:
We can construct an explicit homotopy to one of the boundary circles, namely the map F : (S
1
I) I S
1
I
dened by F(, s, t) = ( + 2ts, s). This is certainly continuous and F(, s, 0) = 1l
S
1
I
, F(, s, 1) = f(, s). Note
3
Math 6510
Professor Hatcher
Homework 1
Net ID: tc328
Tarun Chitra
February 3, 2011
that this homotopy only restricts to the identity, t I on S
1
0. Now we need to show that there doesnt exist
a homotopy H : (S
1
I) I S
1
I that restricts to the identity on S
1
1 S
1
0 for all t I. Suppose
that such a homotopy existed. Now x
0
[0, 2) and consider the set K
t
:= H
t
(
0
, s) : s [0, 1]. Now note that
K
0
= f(
0
, s) : s [0, 1] = (
0
+ 2s, s) : s [0, 1] which is simply the image of f. Let p
1
: S
1
I S
1
be the
projection onto the rst coordinate. Then the set (p
1
(k), 0) : k K
0
is homeomorphic to [
1
]
1
(S
1
0, x
0
), the
loop that wraps around S
1
once. This can be seen in the above image, if you simply project the helix down onto the
base S
1
. Now note that the set K
1
= (
0
, s) : s [0, 1] is simply a line on the cylinder. Projecting this down onto
the base S
1
yields a constant loop, [
0
]
1
(S
1
0, x
0
). This implies that H is a homotopy between [
0
] and [
1
] a
contradiction.
8
No, the Borsuk-Ulam Theorem does not hold for T
2
= S
1
S
1
. We can think of S
1
S
1
C
2
, dened by S
1
S
1
=
(z
1
, z
2
) C
2
: [z
1
[ = [z
2
[ = 1 = (e
2i1
, e
2i2
) C
2
:
1
,
2
[0, 1). Now let n N be odd and consider the map
(e
2i1
, e
2i2
) (
n
1
,
n
2
). The inversions z
1
z
1
, z
2
z
2
send
1

1
+
1
2
,
2

2
+
1
2
. If this map were to
satisfy f(x, y) = f(x, y), then we would have
n
1
= (
1
+
1
2
)
n
,
n
2
= (
2
+
1
2
)
n
. This means that we have to show that
the polynomials,
(
1
+)
n

n
1
=
n1

i=0

n
i

i
1

1
2

ni
(4)
(
2
+)
n

n
2
=
n1

i=0

n
i

i
2

1
2

ni
(5)
dont have any roots in [0, 1). For n = 3 this is true, since we get a quadratic with only imaginary roots. However, note
that it is not possible for a root of either polynomial to be in [0, 1), since we would have a nite sum of strictly positive
numbers. Hence we have an innite sequence of polynomials that violate the Borsuk-Ulam Theorem
9
We want to prove the Ham-Sandwich Theorem for R
3
: If A
1
, A
2
, A
3
are compact sets in R
3
, then a plane that
simultaneously divides the three sets into subsets of equal measure. Without the loss of generality, assume that there
exists a neighborhood U of 0 R
3
such that A
i
U = . By transforming coordinates with an ane translation, we
can always ensure that this happens. First, lets reduce the problem to considering points on the unit sphere, S
2
R
3
.
For each point in a
i
A
i
(for any i), there exists a unique line x
i
connecting a
i
and 0 R
3
. A standard fact about 3-
dimensional geometry is that it takes 3 points (or vectors) to determine a plane up to origin (i.e. to specify an ane plane
that is homeomorphic to the 2-dimensional Ane Space A
2
). Since we have three sets, a point (a
1
, a
2
, a
3
) (A
1
, A
2
, A
3
)
species (up to location of origin) a unique plane passing through the points a
1
, a
2
, a
3
. Since each point corresponds to a
line, we can map a
i
to x
i
S
2
. More precisely, we construct a map : R
3
0 S
2
, (x) = [x], where the equivalence
relation is x y x = y, > 0. This is similar to the construction of RP
2
except that we are looking at equivalence
classes of rays. Note that this map is continuous (it is the retraction of R
3
0 onto S
2
) and one of the maps of a
homotopy equivalence, so that in particular, (A
i
) S
2
is compact. Thus we can specify a plane that intersects all three
sets by specifying points ( a
1
, a
2
, a
3
) (A
1
) (A
2
) (A
3
), up to translation. Since we are measure "equal division"
of the sets A
1
, A
2
, A
3
by using the Lebesgue measure m on R
3
(which is the unique, translational-invariant, complete
Radon measure on R
3
), knowing a plane "up to translation" will not change our results. Another elementary fact from
geometry
2
is that a plane in R
3
divides R
3
into two disjoint subsets R
+
and R

. Given a point ( a
1
, a
2
, a
3
), denote the
partition due to the plane containing (a
1
, a
2
, a
3
) by R
+
( a
1
, a
2
, a
3
), R

( a
1
, a
2
, a
3
). Now suppose for a contradiction
that there does not exist a plane that divides A
i
, i. Then this means that there is no point ( a
1
, a
2
, a
3
) S
2
such
that m(A
i
R
+
( a
1
, a
2
, a
3
)) = m(A
i
R

( a
1
, a
2
, a
3
)). For brevity, let m

i
( a
1
, a
2
, a
3
) := m(A
i
R
+
( a
1
, a
2
, a
3
))
m(A
i
R

( a
1
, a
2
, a
3
)) , m

= m

. Now consider the function, f : S


2
S
2
S
2
R
2
R
2
R
2
dened by:
f( a
1
, a
2
, a
3
) =

1
( a
1
, a
2
, a
3
), m

1
( a
1
, a
2
, a
3
)

2
( a
1
, a
2
, a
3
), m

2
( a
1
, a
2
, a
3
)

3
( a
1
, a
2
, a
3
), m

3
( a
1
, a
2
, a
3
)

(6)
This map is continuous, since we are composing a restriction morphism with the Lebesgue measure yielding a measure
that is absolutely continuous (and has compact support) with respect to m. The projection p
i
f of f onto its ith
coordinate can be considered a continuous map S
2
R
2
so that by the Borsuk-Ulam theorem, there exists x S
2
such that f(x) = f(x). This implies that m

i
(x) = m

i
(x), m

i
= m

i
(x). The equations are redundant and we
2
Unlike the Jordan Curve Theorem!
4
Math 6510
Professor Hatcher
Homework 1
Net ID: tc328
Tarun Chitra
February 3, 2011
only need to consider, without the loss of generality, the case. Since each point x S
2
maps to the intersection of
two open sets this gives m
+
i
(x) m

i
(x) = m
+
i
(x) m

i
(x). As x ,= xx S
2
, the -additivity of the Lebesgue
measure gives m
+
(x (x)) = m
+
(x) m
+
(x) = m

(x) m

(x) = m

(x (x)). This implies that a point


where m
+
= m

, contradicting our assumption that no such point exists. Tracing back through the reductions and
reformulation of this problem as a map on the sphere, this means that there must exist a plane that divides each A
i
.
5

You might also like